1986 AJHSME Problems/Problem 14

Revision as of 10:09, 19 January 2009 by 5849206328x (talk | contribs) (New page: ==Problem== If <math>200\leq a \leq 400</math> and <math>600\leq b\leq 1200</math>, then the largest value of the quotient <math>\frac{b}{a}</math> is <math>\text{(A)}\ \frac{3}{2} \qqua...)
(diff) ← Older revision | Latest revision (diff) | Newer revision → (diff)

Problem

If $200\leq a \leq 400$ and $600\leq b\leq 1200$, then the largest value of the quotient $\frac{b}{a}$ is

$\text{(A)}\ \frac{3}{2} \qquad \text{(B)}\ 3 \qquad \text{(C)}\ 6 \qquad \text{(D)}\ 300 \qquad \text{(E)}\ 600$

Solution

This problem needs a solution. If you have a solution for it, please help us out by adding it.

See Also

1986 AJHSME Problems